SlideShare a Scribd company logo
1 of 15
Download to read offline
MOCK CAT

         VERBAL ABILITY AND READING COMPREHENSION

DIRECTIONS for Questions 1 & 2: Read the arguments and answer the questions that follow.

1. Housing property rentals in city A should go up even more. This will attract more landlords to rent
   out their houses, which in turn will improve the availability of houses in city A. Thus, bringing
   down the housing property rentals in the long run.
   Of the following, which is the assumption made in the above paragraph?

   (1) Rents are low at present.
   (2) Landlords are interested only in rents.
   (3) Tenants are willing to pay more.
   (4) In the long run, the number of houses available for rent will exceed the number of people
   looking for houses.
   (5) Managing property rentals is an organized activity in city A.

2. Children who consistently learn music or painting at an early age demonstrate a deeper
   understanding of their environment and are able to cope with their peers and seniors more amicably.
   Due to this, educators believe that getting children to learn music or painting at an early age will
   increase and hone their social skills.
   Which of the following statements shows a flaw in the reasoning presented above?

   (1) A child’s social skills usually develop at an early stage and remain fixed into his personality.
   (2) Development of social skills depends on the exposure that a child receives through interacting
       and conversing with his parents, siblings, relatives and friends.
   (3) Social skills are essential for any child as they are also responsible for shaping the personality of
   the child.
   (4) Peers and seniors can also help students who feel shy or uncomfortable while interacting with
       other at school or outside of it.
   (5) Children, who possess good social skills go for learning music and painting.

DIRECTIONS for Questions 3 & 4: Each of the following questions has a paragraph from which the
first sentence has been deleted. From the given options choose the one that begins the paragraph in the
most appropriate way.

3. It was the impact of personal events that forced me to translate everything on paper. It was anger,
   jealousy and bitterness that shaped my narration and sketched extremely complex characters. I
   shared my pain with them. As my real life situation became more complex, the suffering and
                      www.zioneducationonline.com                                    9899758900, 9891638000
                          Leading institute for MBA/GMAT/GRE/BBA/BCA/Hotel Mgt/Law entrance exams
                 Centres at: North Campus, Pitampura, Rajinder Nagar, Noida, Greater Noida, Indirapuram(Ghaziabad)
bitterness of my characters increased. Some of them became real monsters, while others showed
   fewer signs of sanity. I was amazed at the powerful creations that my emotions had unleashed. Not
   only had my emotions helped to create a world of my own; they had made me a writer as well.

   (1) Earlier when I wrote, I undermined the power of emotions and preferred to be inspired by facts.
   (2) When I was disgusted or hurt, I wrote.
   (3) Writing style truly represents the source of a writer’s inspiration and ideas.
   (4) My negative experiences translated into vicious characters on paper.
   (5) I am a victim of my emotions.

4. Owned by the Wrigleys, this magnificent piece of art had walls made from Portland stone. The open
   lobby was about 40 feet by 32 feet with four pillars that supported the first floor. Fifteen bedrooms
   containing unique mahogany wood work, five splendid dining areas and a picturesque lawn adorned
   this property. It was an unknown architect’s brilliant creation that displayed a refined taste and
   superior technical knowledge.
   (1) For centuries, the Wrigley family had bought numerous properties all over the world.
   (2) The Wrigley’s mansion was one of the best properties in this area.
   (3) The mansion was not too different from any others in this part of the country.
   (4) Its architectural beauty had drawn many admirers from all over the world.
   (5) The house was , if it could be called one, was magnificient.

DIRECTIONS for Questions 5 to 7: Read the passage given below and answer the questions that
   follow.

“A boy is not happy owing to his age; boys who are called happy are being congratulated by reason of
the hopes we have for them. For there is required not only complete virtue, but also a complete life,
since many changes occur in life, and all manner of chances, and the most prosperous may fall into
great misfortunes in old age.”

In other words, what Aristotle is saying is that what is required for happiness is “a complete life” which
obviously no young person has while he is still young. He makes the same point in another way. He
refers to the story of Croesus and Solon, as told by the ancient Greek historian, Herodotus. Croesus was
King of Lydia, and one of the richest and most powerful rulers of his day. Solon was one of the wisest
men of Greece. Here is the story of their conversation.

“Solon set out upon his travels, in the course of which he came on a visit to Croesus at Sardis. Croesus
received him as his guest, and lodged him in the royal palace, and had his servants conduct him over his
treasures, and show him all their greatness and magnificence. And when Solon had seen them all.
Croesus said, ‘Stranger of Athens, I have heard much of your wisdom and of your travels through many
lands. I am curious therefore to ask you, whom of all the men that you have seen, you consider the most
happy?’ This he asked because he thought himself the happiest of mortals: but Solon answered him
without flattery: ‘Tell us of Athens, sire.’ Astonished at what he heard, Croesus demanded sharply,
‘And why do you consider Tell us the happiest of men?’ To which the other replied, ‘First because his
country was flourishing in his days, and he himself had sons both beautiful and good, and he lived to
see children born to each of them, and these children all grew up; and further because, after a life spent
in what our people look upon as comfort his end was glorious. In a battle between tie Athenians and
their neighbors near Eleusis, he died gallantly upon the field. And the Athenians gave him a public
funeral and paid him the highest honors.” Thus, Solon admonished Croesus by the example of Tell us.
                       www.zioneducationonline.com                                    9899758900, 9891638000
                           Leading institute for MBA/GMAT/GRE/BBA/BCA/Hotel Mgt/Law entrance exams
                  Centres at: North Campus, Pitampura, Rajinder Nagar, Noida, Greater Noida, Indirapuram(Ghaziabad)
When he had ended, Croesus asked angrily, ‘Is my happiness, then, so little to you that you do not even
put me on a level with private men?’

5. It can be inferred from the passage that according to Aristotle

   (1)   Young people cannot be happy.
   (2)   Young people lack complete virtue.
   (3)   Young people can be happy if they have children.
   (4)   There are many factors which need to be fulfilled for a young person to be happy.
   (5)   Young people can be happy if they have high hopes.

6. Which of the following is most likely to follow the passage as Solon’s answer to Croesus?

   (1) ‘I see that You are wonderfully rich and are the lord of many nations, and as for your question, I
       would like to hear of the wealth that you have amassed’.
   (2) ‘I see that You are wonderfully rich and are the lord of many nations, but as for your question, I
       have no answer to give until I hear that you have closed your life happily’
   (3) ‘I see that You are wonderfully rich and are the lord of many nations, but as for your question, I
       have no answer to give until I hear about your children and grandchildren’
   (4) ‘I see that You are wonderfully rich and are the lord of many nations, but as for your question, I
       have no answer to give until I hear that you have led a virtuous life’.
   (5) ‘I see that You are wonderfully rich and are the lord of many nations, but as for your question, I
       have no answer to give until I hear how you have faced misfortune’.

7. A tone /emotion which is not part of the passage would be?

   (1) Rebuke            (2) Rhetorical                  (3) Narration                   4) Didactic                   (5) Pity

DIRECTIONS for Questions 8 to 10: Arrange the sentences A, B, C and D to form a logical sequence
between sentences 1 and 6.

8. 1. Meeting people after nine to ten years, almost to the day, is a very weird experience.
   A. It genuinely felt awkward to meet people, some married and some with children, others married
      but who forgot to send out 200 cards to school friends, others, divorced.
   B. I did find out that the marriage was unhappy for all the wrong reasons, none because of the
      obnoxious twit that he is.
   C. I actually felt sorry for one of those guys, because, and if you knew equation with him in the
      school bus where we almost killed each other a few times, I would not have wished a divorce on
      him -though, I would not have wished any woman on him either.
   D. I am in close touch with a couple of school friends - Doc, for example is an ass I can still call
      my best friend after twenty years - but my god, did he (or rather his overheating BMW) push my
      patience on Saturday night.
   6. Everybody was fatter/ balder and in some cases both.

   (1) DCBA              (2) CBDA                        (3) DACB                        (4) BCDA                      (5) ABCD

9. 1. Even though dance starts a bit later than school does during the week, Saturday morning is still
      pretty chaotic around here.
                        www.zioneducationonline.com                                    9899758900, 9891638000
                            Leading institute for MBA/GMAT/GRE/BBA/BCA/Hotel Mgt/Law entrance exams
                   Centres at: North Campus, Pitampura, Rajinder Nagar, Noida, Greater Noida, Indirapuram(Ghaziabad)
A. More so if John is leaving for an auction that day, because it means I have to truck Kristen and
         Alex with me and get them ready too.
      B. This is one of those mornings where John was rushing to leave too.
      C. There are buns to be done, bodysuits to find, tights to mend (because they’re always ripped
         somewhere) and a good breakfast to be had.
      D. His work van has been giving him some problems, so he was nervous about travelling with it
         (not to mention what it’s costing us to fix it).
      6. He couldn’t find his cell and even though he had woken up in a general good mood, I could see
         it going downhill from there.

      (1) ACBD               (2) ABCD                        (3) ADCB                        (4) BCDA                      (5) BDCA

10. 1. Climate change will trigger a chain of events that is likely to prompt an increase in HIV rates
       world wide, an expert has warned.
    A. Daniel Tarantola of the University of New South Wales (UNSW) said the disadvantage in
       developing countries must be addressed if the world is to prevent a dramatic escalation of the
       HIV epidemic as well as other health problems.
    B. "It was clear soon after the emergence of the HIV epidemic that discrimination, gender
       inequality and lack of access to essential services have made some populations more vulnerable
       than others, "Tarantola said on Wednesday.
    C. "Today, additional threats are lurking on the horizon as the global economic situation
       deteriorates, food scarcity worsens and climate change begins to affect those who were already
       dependent on survival economies," Tarantola said.
    D. David Cooper, also of UNSW, said: “Science has achieved great strides towards shaping a more
       effective response to HIV.
    6. Yet research has not succeeded in producing the hoped-for 'magic bullets' of either a cure or a
       vaccine.”

      (1) BACD               (2) DCBA                        (3) CDBA                        (4) ABCD                      (5) ADCB

DIRECTIONS for questions 11 to 14: Fill in the blanks with the correct option given below:

11. Rich in local color, the undercurrents that run beneath the seemingly ………… surroundings of the
   sleepy village are explored in fluid prose.

      (1) pristine                      (2) idyllic                     (3) clarity                    (4) liquid          (5) lazy

12.       With models and beauty queens becoming ……..for the youth, fashion is a …… sector.

(1) icons; thriving          (2) idols; recommended                     (3) ideals; booming            (4) None of these              (5)
mentors; slowing

13. Leaving the country of one's ……… for a new land is a major decision but globalization has taken
   away the fear and lessened the anxiety.

(1) growth           (2) death                    (3) birth                       (4) progress         (5) family


                            www.zioneducationonline.com                                    9899758900, 9891638000
                                Leading institute for MBA/GMAT/GRE/BBA/BCA/Hotel Mgt/Law entrance exams
                       Centres at: North Campus, Pitampura, Rajinder Nagar, Noida, Greater Noida, Indirapuram(Ghaziabad)
14. As the second generation of the post 1965 Indian immigrants come of age in America, we are
    beginning to see a wealth of literature as well as critical essays in which they …………… and
    …………. their experiences.

(1) chronicle; analyse             (2) extend; elaborate (3) list; expunge                        (4) sort; practice   (5)
decry ;dissuade

DIRECTIONS for questions 15 to 17: Read the passage and answer whether the statements that
follow are:
       (A) a logical conclusion to the passage.
       (B) a contradiction to the intent of the passage.
       (C) a far-fetched conclusion.
       (D) Irrelevant to the passage.


Passage
Ramesh Melwani joined a leading leather company. An entrepreneur-driven, fast growing, exporter of
leather accessories, the company had been doubling its turnover every year for the last six years.
Melwani joined with a 2.5-fold jump in salary. Within a week he was back in his old company. Reason:
he couldn't survive the ruthless, 16-hour-a-day regimen, where the individual was given no respect.
Today he's a lot poorer, a lot happier.

15. High salaries do not necessarily motivate or help retain talent in an organisation.


16. The entrepreneur alone can make or break an organisation.


17. The irrevocable bottom line in the new age organisations is human resource development.


DIRECTIONS for question 18 Relate to the following passage and answer the question:

An important question remains: Why has atheism and skepticism toward religion suddenly emerged as
a question of great current interest, at least among the literate, in the past few years? Clearly something
has happened to break atheists of their tendency to nurture their disbelief privately and to keep their
opinions to themselves.

It seems obvious that politics has a lot to do with this, specifically the cultivation of the religious right
as a phalanx of conservative storm-troopers who are rewarded by conservative politicians by having
their singular dogmas incorporated, as much as possible, into public policy. The increasing pressure on
women’s reproductive rights, the suppression of stem cell research and, most egregiously, the fresh
intrusion of creationism into public schools are primary instances of this. Beyond these concrete
horrors, there is no escaping the fact that the miasma of compulsory religiosity has thickened and
diffused throughout society. For instance, one notes, rather queasily, the success of the Evangelicals in
turning the Air Force Academy into a virtual fundamentalist seminary where cadets from all sorts of
backgrounds are relentlessly pressured by officers and upperclassmen into declaring for the Born-
Agains.
                       www.zioneducationonline.com                                    9899758900, 9891638000
                           Leading institute for MBA/GMAT/GRE/BBA/BCA/Hotel Mgt/Law entrance exams
                  Centres at: North Campus, Pitampura, Rajinder Nagar, Noida, Greater Noida, Indirapuram(Ghaziabad)
Atheists, who, despite polls, have never been all that rare, have come to mistrust the notion that they
can believe as they will, undisturbed, provided they remain discreet about it. The mood fostered by the
religious right seems to tend toward the inquisitorial. Scientists, in particular, representing the one
vocation in which non-belief is the norm, rather than the outlier, have sensibly concluded that the
culture in which they have quietly lived is being attacked at its foundations. It’s one thing to send your
kids to a public school where “under god” is formulaically recited as part of the daily Pledge of
Allegiance. It’s quite another to have your kid’s elementary biology class interrupted by harangues
against ‘Darwinism’, or to see the Bible, taught as literal truth, surreptitiously introduced into the
curriculum. When matter shave come to that pass, scientists, among others, see little point in not
fighting back openly. Thus, one now sees a torrent of books, largely by scientists and sympathetic
philosophers, striking back not only at the enemies of stem-cell research and the proponents of
Intelligent Design Theory, but at the very roots of the cultural tic that provides these miscreants such
fertile ground: supernatural religion predicated upon a supreme being.

Then, too, one must consider the possibility that, as the ostensible religious uniformity of the nation
begins to dissolve, the sort of person who habitually responds to questionnaires about faith by listing
himself as ‘unaffiliated, but a believer’ has, at long last, become more honest with himself and with the
rest of us, frankly acknowledging the deep vein of skepticism underlying his disinclination to become a
churchgoer. I suspect that many of the readers of the New Atheists’ books fall into this category, and
are more willing to let their doubts see the light of day.

18. In the light of your reading of the passage above, identify the option that contains the set of words
    CLOSEST in the meaning to the words in CAPITALS:
    EGREGIOUS: SKEPTICISM: INQUISITORIAL

   (A) blatant: suspicion : intolerant                  (B) intolerant : certitude : keen
   (C) balanced: confident : inane                              (D) mean : interrogation : impatient
   (E) unsure: bored: crazy

DIRECTIONS for Question 19: Each of the following questions has a paragraph from which the last
sentence                          has                           been                          deleted.
From the given options, choose the one that completes the paragraph in the most appropriate way.

19. The difference between the revolutionary and the terrorist lies in the reason for which each fights.
    For whoever stands by a just cause and fights for the freedom and liberation of his land from the
    invaders, the settlers and the colonialists cannot possibly be called terrorist, otherwise the American
    people in their struggle for liberation from the British colonialists would have been terrorists the
    European resistance against the Nazis would be terrorism, the struggle of the Asian, African and
    Latin American peoples would also be terrorism, and many of you who are in this Assembly hall
    were considered terrorists. This is actually a just and proper struggle consecrated by the United
    Nations Charter and by the Universal Declaration of Human Rights. As to those who fight against
    the just causes, those who wage war to occupy, colonize and oppress other people, those are the
    terrorists.

   (1) They are the ones who invade everywhere; they are the ones with no clear goal.
   (2) Because the justice of the cause determines the right to struggle.
   (3) The end is as important as the means.
                       www.zioneducationonline.com                                    9899758900, 9891638000
                           Leading institute for MBA/GMAT/GRE/BBA/BCA/Hotel Mgt/Law entrance exams
                  Centres at: North Campus, Pitampura, Rajinder Nagar, Noida, Greater Noida, Indirapuram(Ghaziabad)
(4) I urge everyone present here to resound this with me that the people of Palestine are not terrorist
s.
     (5) Not the Asians, not the Africans and not the Latin Americans for sure.


DIRECTIONS for Question 20 to 22: Each of the following passages are followed by an inference or
inferences. Examine each inference separately in the context of the passage. Each inference is denoted
by:
       1 if the inference is definitely true, i.e. it directly follows from the statement of facts given.
       2 if the inference is probably true, though not directly true, in the light of the statement of facts
       given.
       3 if the inference is uncertain, i.e. data is insufficient to decide whether the inference is true or
       false.
       4 if the inference is probably false, though not definitely false, in the light of the statement of
       facts given.
       5 if the inference is definitely false, i.e. it cannot possibly be inferred from the statement of facts
       given.
Choose the option that gives the most appropriate set of conclusions about the inference or inferences.

20. Native Chinese geography begins in the Warring States period (5th century BC). It expands its
    scope beyond the Chinese homeland with the growth of the Chinese Empire under the Han Dynasty.
    It enters its golden age with the invention of the compass in the 11th century (Song Dynasty) and
    peaks with 15th century (Ming Dynasty) Chinese exploration of the Pacific under admiral Zheng
    He.
    Inference: China was ruled by the Han dynasty at some point of time in its history.

     (1) Option 1         (2) Option 2                    (3) Option 3                    (4) Option 4                  (5) Option 5



21. Pop music is an ample and imprecise category of modern music not defined by artistic
    considerations but by its potential audience or prospective market. Pop is music composed with
    deliberate intent to appeal to the majority of its contemporaries.
    Inference: Pop music is more commercial in nature than all other forms of music.

     (1) Option 1         (2) Option 2                    (3) Option 3                    (4) Option 4                  (5) Option 5

22. A compass (or mariner compass) is a navigational instrument for finding directions on the Earth. It
    consists of a magnetized pointer free to align itself accurately with Earth's magnetic field, which is
    of great assistance in navigation.
    Inference: Most modern ships use a compass to navigate their way around oceans.

     (1) Option 1         (2) Option 2                    (3) Option 3                    (4) Option 4                  (5) Option 5



     DIRECTIONS for Question 23 to 25:Each of the questions below has a set of sequentially
     ordered statements. Each statement can be classified as one of the following:


                         www.zioneducationonline.com                                    9899758900, 9891638000
                             Leading institute for MBA/GMAT/GRE/BBA/BCA/Hotel Mgt/Law entrance exams
                    Centres at: North Campus, Pitampura, Rajinder Nagar, Noida, Greater Noida, Indirapuram(Ghaziabad)
A. Facts, which deal with the pieces of information that one has heard, seen or read, and which
       are open to discovery or verification (the answer option indicates such a statement with an
       ‘F’)
    B. Inferences, which are conclusions drawn about the unknown, on the basis of the known (the
       answer option indicates such a statement with an ‘I’)
    C. Judgements, which are opinions that imply approval or disapproval of persons, objects,
       situations and occurrences in the past, the present or the future (the answer option indicates
       such a statement with a ‘J’)

 Select the answer option that best describes the set of statements.

 23. A. Community is our stickiest glue, and nothing bonds us as willingly as our clan food.
     B. So, even if Bhajji now bats for Mumbai, nothing will ever make us stop rooting for the kind
        of roti we ate in our childhood kitchens.
     C. The handi that rocked our cradle will always rule our world.
     D. Cities also have a culinary DNA, and it is not easy to auction them off to the highest pizza
     franchise.
     E. It is worse for cities which stagger under the burden of their legendary cuisines.

    (1) IJIJI                     (2) JIJIJ                      (3) IIJJI                      (4) JJJJJ               (5) JJIIJ

24. A. No one needs to be reminded of the ferment which is moving in the world of social affairs, of
      the obscure but powerful tendencies which are forcing society out of its grooves and leaving
      it, aspiring but dubious, in new and uncharted regions.
    B. This may affect different minds in different ways.
    C. Time- honoured political and economic formulae are become "old clothes" for an awakened
       and ardent generation, and before the new garments are quite ready the old one’s are shed.
    D. The blessed word "reconstruction" is often mentioned.

    (1) JIJF                      (2) JFFF                       (3) JFJI                       (4) IIJI                (5) JIJI

 25. A. It seems that men are not satisfied that society has really developed so successfully as it
         might have done; many believe that it finds itself in a cul-de-sac.
     B. The experienced can see that many of the offered reforms are but the repetition of old
        mistakes which will involve us in the same unhappy cycle of disillusion and failure.
     C. It is not to be wondered, if men everywhere are seeking for a sign, a glimpse of a scheme of
        life, a view of reality, a hint of human destiny and the true outcome of human effort, to be an
        inspiration and a guide to them in their pathetic struggle out of the morass.
     D. If Philosophy has anything to say which is to the point, then let Philosophy by all means say
        it.
     E. The people may indeed expect too much from it, as those who best grasp the measure of
        Philosophy's task are the first to urge an outcome.

    (1) IFJJF         (2) FFIJJ               (3) IFJJI                      (4) JFJJJ                      (5) FFJJJ


     DATA INTERPRETATION AND LOGICAL REASONING.

                     www.zioneducationonline.com                                    9899758900, 9891638000
                         Leading institute for MBA/GMAT/GRE/BBA/BCA/Hotel Mgt/Law entrance exams
                Centres at: North Campus, Pitampura, Rajinder Nagar, Noida, Greater Noida, Indirapuram(Ghaziabad)
26.    If it is true that ‘Neither sham is true nor it is practical’, then which one of the following
       statements is true?
       (1) Sham is true; however, it is not practical.
       (2) Sham is true and it is practical.
       (3) Sham is practical and untrue.
       (4) Sham is not practical and it is not true.
       (5) None of the above.

27.    Identify the statement which is False, if it is true that ‘no people other than the members of the
       club are allowed inside the club premises’.
       (1) Some people are members of the club.
       (2) All members of the club may be allowed inside the club premises.
       (3) All people are allowed inside the club premises.
       (4) All those who are allowed inside the club premises are members of the club.
       (5) None of the above.

DIRECTIONS for Questions 28 and 31:
In this caselet whole numbers (not fractions of numbers) from 1 to 26 have been replaced by different
letters of the alphabet. The sums below will enable you to crack the code. NB - The only arithmetical
signs used in this puzzle are ‘+’ (plus), ‘–’ (minus) and ‘ × ’ (times).
       1.    A+B=C             9.    R+J=S
       2.    D=E+F            10.    S+T=U
       3.    F × F=G          11.    W=N+Q
       4.    H=I+J            12.    V+V=W
       5.    K=L+M            13.    X ×Y = K
       6.    M × M=N          14.    Y+B=Z
       7.    N+F=O            15.    Z=P+S
       8.    P× Q = R
28.   If 20 ≤ G ≤ 26 then G = ?
      1. 21           2. 27                  3. 25                 4. 36                5. 40
29.   If K = 24 and X is prime then Y = ?
      1. 8            2. 6           3. 12                         4. 24                5. 36
30.   If S = 13 then R = ?
      1.10             2.12                  3. 9                  4. 8                 5. 6
31.   If N is a square of a number which is the square of a prime number then N = ?
      1. 81             2. 16        3. 625          4. 256         5. 125

DIRECTIONS for Questions 32 to 35: Answer the questions on the basis of the information given
below.

Devendra has 15 different locks viz. L1, L2 through L15 The locks were classified into three categories
based upon their number of levers. L1, L15, L13 and L4 were of 8 levers each. L6, L2, L7, L9 and L14
were of6 levers each and the remaining locks were of 10 levers each. He selects five out of these fifteen
                       www.zioneducationonline.com                                    9899758900, 9891638000
                           Leading institute for MBA/GMAT/GRE/BBA/BCA/Hotel Mgt/Law entrance exams
                  Centres at: North Campus, Pitampura, Rajinder Nagar, Noida, Greater Noida, Indirapuram(Ghaziabad)
locks on each of the eight consecutive days viz. Day 1, Day 2, Day 3, Day 4, Day 5, Day 6, Day 7 and
Day 8. On each day he tries to unlock the selected locks but is successful in unlocking only one of the
selected locks. If he unlocks any lock on a particular day, that lock is not selected by him on the
subsequent days. It is also known that out of the locks unlocked by him, the number of locks of 6 levers
is less than the number of locks of 8 levers. The following table gives details about the locks selected
by him on the given six days.

                                 Day 1         L1        L7         L8         L4        L11
                                 Day 2         L3        L15        L14        L12       L8
                                 Day 3         L2        L7         L15        L9        L10
                                 Day 4         L15       L13        L10        L3        L6
                                 Day 5         L2        L15        L9         L10       L5
                                 Day 6         L13       L6         L1         L8        L10
                                 Day 7         L14       L11        L8         L2        L13
                                 Day 8         L5        L6         L10        L14       L11

32.    On how many days it is possible to uniquely determine which lock was unlocked by Devendra?
       (1) Three   (2) Four        (3) Five               (4) Six              (5) Seven

33.    If after day 8 Devendra found that aggregate levers of all the locks that he has unlocked is
       minimum then the lever of the lock unlocked on day 7 is
       (1) 6          (2) 8         (3) 10        (4) Either (1) or (2) (5) Either (1) or (3)

Additional Information for questions 34 & 35: After Day 8 Devendra found that aggregate levers of
all the locks that he has unlocked is more than 64 but not more than 68.

34.    Which of the following locks is definitely unlocked by Devendra?
       (1) L15              (2) L13                 (3) L10             (4) L11                                      (5)
       Cannot be determined

35.    If L9 is not unlocked and L8 is unlocked by Devendra, then which of the following can be the
       lock that is unlocked by him?
       (1) L2          (2) L5        (3) L11            (4) L14               (5) L10



DIRECTIONS for Questions 36 to 39 : Refer to the following data :
In the Institute of Physical Education in Moskwa,                   grades are awarded according to the following
arrangement :
If percentage score is upto 10                  –                   Grade I
If percentage score is more than 10 and upto 25 –                   Grade H
If percentage score is more than 25 and upto 37 –                   Grade G
If percentage score is more than 37 and upto 50 –                   Grade F
If percentage score is more than 50 and upto 60 –                   Grade E
If percentage score is more than 60 and upto 70 –                   Grade D

                      www.zioneducationonline.com                                    9899758900, 9891638000
                          Leading institute for MBA/GMAT/GRE/BBA/BCA/Hotel Mgt/Law entrance exams
                 Centres at: North Campus, Pitampura, Rajinder Nagar, Noida, Greater Noida, Indirapuram(Ghaziabad)
If percentage score is more than 70 and upto 80 –                    Grade C
If percentage score is more than 80 and upto 90 –                    Grade B
If percentage score is more than 90 and upto 100                     –     Grade A
Following table gives raw scores for four students of the institute, stored in a computer. Scores of
Evanova, however, have been lost due to an error.
                              1st Semester 2nd                              3rd                    4th
                                           Semester                         Semester               Semester
               Anne           38                     89                     93                     0
               Bette          51                     71                     65                     21
               Cynthia        71                     28                     70                     10
               Debora         80                     25                     83                     90
               Evanova

It is known that a grade each (out of the four) of Evanova is common with (same as) another student’s
grade (though not necessarily of the same semester). However, one of Cynthia’s grades is the same as
Evanova’s in the same semester. Bette’s two of the grades are same as Evanova’s but in different
semesters. Similarly, Debora’s two of the grades are also same as Evanova’s but in different semesters.
Anne has only one grade common with Evanova (that too in a different semester).
Except as mentioned above, the students have no common grades.
36.   What is the minimum marks that Evanova could have received in any one semester ?
      1. 0            2. 11        3. 26         4. 38         5. 40
37.   Evanova’s grades are same as Cynthia’s in which of the following semesters ?
      1. 1st         2. 2nd         3. 3rd         4. 4th         5. None
38.   If grade F and lower grades are termed ‘fail grades’, how many ‘fail grades’ were awarded to the
      five students during the four semesters ?
      1. 6             2. 7           3. 8           4. 9         5. 10
39.   If it is assumed for want of availability of actual marks that Evanova’s score for any semester is
      mid-value of the class corresponding to the grade, her cumulative score for the four semesters was
      nearest to which other student’s score ?
      1. Anne          2. Bette               3. Cynthia     4. Debora     5. Anne & Bette both

DIRECTIONS for 40 to 43: Answer each of these questions independently:

40.    From a bag containing 100 balls, one ball weighs 9 grams and all the other weigh 10 grams
       each. Using a simple balance where balls can be kept on either pan, what is the minimum
       weighs required to identify the defective ball?
       (1) 3         (2) 4           (3) 5           (4) 7    (5) Cannot be determined



                       www.zioneducationonline.com                                    9899758900, 9891638000
                           Leading institute for MBA/GMAT/GRE/BBA/BCA/Hotel Mgt/Law entrance exams
                  Centres at: North Campus, Pitampura, Rajinder Nagar, Noida, Greater Noida, Indirapuram(Ghaziabad)
41.            Blue ties go only with yellow or brown jackets. Red ties go only with brown or blue
       jackets. White shoes go only with blue ties or brown jackets. Which of the following is a valid
       dressing combination?
       (1) White shoes, Brown jackets and Blue ties.
       (2) Brown jacket, Red tie and White shoes.
       (3) White shoes, Blue jacket and Red ties.
       (4) Blue Jacket, White Tie, Red Shoes
       (5) All of the above.


42.           4 books were distributed among 3 friends, such that each received atleast 1 book. X
       received Ayn Rand’s Fountainhead. Y did not receive any Ayn Rand book. Z did not receive
       Stephen Covey’s 7 habits or Shiv Khera’s Born to win. If the fourth book was Ayn Rand’s
       Night of January 16th, who received the 7 habits?
       (1) X         (2) Y          (3) Z                (4) Indeterminate (5) None


43.           There are 100 statements written on the board. Statement 1 is “only 1 statement among
       the 100 is false”. Statement 2 is “ only 2 statements among the 100 are false”. Hence each nth
       statement reads “Only n statements among the 100 are false”. How many statements on the
       board are true if the last statement is “Only 100 statements among the 100 are false”?
       (1) 0           (2) 1            (3) 50                (4) Indeterminate (5) 99


DIRECTIONS for questions 44 & 45: Refer to the data below.
Vibha has a choice of the following fruits for dinner - oranges, mangoes, bananas, pineapple, peaches,
apples, guavas, and sweet lime. Given below is some additional information: Oranges and sweet lime
are citrous fruits. Vibha has four fruits for dinner on any given day. Vibha does not have citrous fruits
along with mangoes. Vibha eats peaches only if she eats sweet lime. Vibha has at least one of the three
fruits - bananas, guavas, apples - everyday. Only one unit of any fruit can be eaten.

44.   If Vibha eats peaches, which of the following fruits can she not eat on the same day?
      (1) Banana       (2) Guava      (3) Mango      (4) Orange     (5) Apples

45.   Vibha eats sweet lime and guavas, then the number of possible combinations is
      (1) 10          (2) 14        (3) 20         (4) 16         (5) 24

                                                PROBLEM SOLVING

Q46. Bhuvanesh alone can build a wall in 15 days while Kalpesh can demolish the same by himself in
25 days. If Bhuvanesh starts working on odd date, Gurneet helps him. Ivneet helps Kalpesh only if
Gurneet helps Bhuvanesh. In a day, Gurneet does half as much work as Kalpesh and Ivneet does half as
much work as Bhuvanesh. Find the minimum number of days required to build the wall if Bhuvanesh
abd Kalpesh work on alternate days?
   1) 150            2) 75          3) 151        4) 201        5) None of these

Q47. The hundred’s place digit of (251)^148 is
   1) 0      2) 1 3) 5 4) 4
                       www.zioneducationonline.com                                    9899758900, 9891638000
                           Leading institute for MBA/GMAT/GRE/BBA/BCA/Hotel Mgt/Law entrance exams
                  Centres at: North Campus, Pitampura, Rajinder Nagar, Noida, Greater Noida, Indirapuram(Ghaziabad)
Q48. In a cricket match between India and Pakistan, 5 batsmen scored centuries. If 11 batsmen from
each team batted, how many different sets of batsmen could have scored centuries?
   1) 25132          2) 26434       3) 26334       4) 29262       5) None of these

Q49. Of all the dresses she has, Kiza wears only the Red one, the green one and the Blue one to office.
She dresses up in the following manner: She does not wear Red dress on any 2 consecutive days. She
wears Green dress only if she has worn the blue dress on the previous day. In how many ways can she
dress up for office on the 5 working days Monday to Friday, if she wants to wear the red dress at least
once?
    1) 36              2) 33         3) 45          4) 21 5) None of these

Q50. 6 countries send their Prime ministers and Presidents for a UN conclave. In a round of 6
discussions, held at the conclave, each discussion was between a Prime minister and a President both
from different countries. Find the total number of ways in which this conclave could have been
conducted?
    1) 984             2) 265        3) 1412              4) 1628              4) None of these.

Q51. Ankit and Arpit started a cycle race from A to reach B. Arpit cycled at ¾th of Ankit’s speed.
Ankit broke some rules and as a penalty had to go back to A after covering half the distance and then
resumed the race. How long did Arpit take to finish the race if he won by 10 minutes?
    1) 20 min        2) 25 min      3) 45 min       4) 80 min

Directions for Ques 7 and 8. In a survey conducted on a group of 366 women over what they like about
men, it was found that out of the total women who liked men for their sense of humour, 30% liked men
with a good bank balance. A total of 90 women liked men for their bank balance only and the number
of women liking men for their sense of humour and good looks was same as the number of women
liking men for their good looks and bank balance. The ratio of the number of women liking men for
their sense of humour and for their good looks was 2:3. No women liked men for all the 3 qualities.
Q52. If the number of women liking men for their bank balance and sense of humour was 1.5 times the
number of women liking men for their sense of humour and good looks, what was the number of
women who liked men for their good looks only?
    1) 86             2) 100          3) 120        4) 132        5) None of these.

Q53. If the number of women who liked men only for their sense of humour was 24, what was the
number of women who liked men for their good looks and bank balance?
   1) 73            2) 74        3) 265         4) Cannot be determined  5) None of these.

Q54. (100)! Is written in base 21. How many Zeroes will it have on the right of 1st non-zero digit from
the right?
    1) 24      2) 48          3) 12       3) 14           5) 16

Q55. Zion Education, a CAT coaching institute, has a weird teacher, who teaches Maths through
games. He writes two different linear expressions on 2 chits, namely (7 – x) and ( 3 + x). Everyday he
asks 2 students to chose one chit each and asks a third one to assign an integral value to the variable x.
The student who gets the maximum value for the expression on his chit, for this value of ‘x’, gets a
number of chocolates equal to the value of the expression. What is the minimum number of chocolates
the teacher would have to give away at the end of 7 days, if the 3rd student assigns a different value to x
everyday?
    1) 35      2) 47          3) 56          4) 21          5) None of these
                       www.zioneducationonline.com                                    9899758900, 9891638000
                           Leading institute for MBA/GMAT/GRE/BBA/BCA/Hotel Mgt/Law entrance exams
                  Centres at: North Campus, Pitampura, Rajinder Nagar, Noida, Greater Noida, Indirapuram(Ghaziabad)
Q56. 3 friends Magnum, Maggi and Maggu rolled a dice 3 times each. The outcomes were such that:
   i) For each of them, the 3 outcomes were distinct.

   ii) For all of them, the sum of the outcomes was the same and was more than 10.

   iii) Only one outcome was the same between any two of the 3 friends.

Which of the following cannot be the common outcome between any 2 of the 3 friends?
  1) 4               2) 2           3) 6         4) 3         5) None of these.

Q57. Rituanjay marks up his goods by x% and then gives a discount of y% and thus makes a profit of
53%. If he had marked his goods up by y% and then given a discount of x%, then he would have
suffered a loss of 77%. What is the value of x? The cost price of goods is Rs.100
    1) 45             2) 15           3) 80          4) Cannot be determined      5) None of these.

Q58. 2 sqaures are selected at random on a 8 x 8 chess board. What is the probability that they have a
side in common.
    1) 1/6            2) 1/12       3) 1/18        4) 1/9       5) It’s not possible

Q59. A merchant navy ship with 9 different holes vertically aligned in its hull springs a leak in any of
it’s 3 consecutive holes if 3 consecutive holes are closed. Find the number of ways in which it can
spring a leak.
     1) 10            2) 18         3) 17          4) 20          5) None of these

  Q60. Kiza and Fiza are twins. They are going for their CAT classes at Zion Education, which is 20
kms away from their home. They have 1 bicycle but they hate travelling with each other as people
misunderstand Kiza for Fiza and Fiza for Kiza. They decide that Kiza will walk first and Fiza will go
by bicycle and will leave at Delhi University Metro Station, where from Kiza will pick the bicycle and
will go to Zion. Kiza walks at 5 k/h and cycles at 12 k/h while Fiza walks at 4 k/h and cycles at 10 k/h.
They started from their house at the same time. How far should be Delhi University Metro Station from
their house given that Kiza and Fiza reach Zion at the same time.

   1) 7.5 km           2) 10 km             3) 12.5 km            4) 14.5 km           5) Data Insufficient

Q61. 2 cards are drawn at random from a standard pack of 52 cards. What is the probability that 1 of the
cards is King and another Hearts?
    1) 1/13           2) C(4, 1) * C(13, 1) / C(52, 2)    3) 1/26        4) Cannot be determined
               5) None of these

Q62. Given that LogX (LogY (Logz p))) = 0, where each of x, y and z can assume values among 3, 27
and 81 only. If the product of all possible values of ‘p’ is represented in the form of 3^n, then what is
the value of ‘n’?

   1) 400              2) 480               3) 520                4) 360               4) 380

Q63. In a regular hexagon having a diagonal of length 56 cm, how many equilateral triangles of side 4
cm can one fit provided no triangle will overlap or intersect with each other?

                      www.zioneducationonline.com                                    9899758900, 9891638000
                          Leading institute for MBA/GMAT/GRE/BBA/BCA/Hotel Mgt/Law entrance exams
                 Centres at: North Campus, Pitampura, Rajinder Nagar, Noida, Greater Noida, Indirapuram(Ghaziabad)
1) 294              2) 60                3) 90                 4) 45                5) None of these

Q64. 2^100 – 1 is written in base 2. How many 1’s will it have?
   1) 95      2) 100          3) 101       4) 105           5) None of these

Q65. D, E and F are the points on sides BC, CA and AB of triangle ABC such that BD = DC; CE = 2
EA; AF = 3 FB. A(Tri DEF) =
   1) ½ A( ABC)             2) ¾ A( ABC)         3) 7/24 A( ABC)      4) ½ A( ABC)

Q66. Kripa singh uses a faulty weighing machine. The left pan of his beam balance is 200 gms heavier
then the right pan. He uses the same beam balance while buying as well as selling. Assuming that he
uses the beam balance logically, what is his overall profit percent.
    1) 16.66%                 2) 20%          3) 44.44%       4) 50%    5) cannot be determined

67.        If N = (2012 + 201) – (1992 + 199), then what is the remainder when N is divided by 399?
      1. 0                2. 2                   3. 4                4. 399            5. None of these

68.                                           [(            )]
           What is the remainder when 257 + 338 2 is divided by 17?
      1. 0                2. 1                3. 2                4. 3                                            5. 4

69.      If any 4 whole numbers are chosen randomly and multiplied to each other, then what is the
      probability that the last digit of the product thus obtained is 1, 3, 7 or 9?
               4                    4                                4                             5
         2                 1                              1                        1 2
      1.                2.                            3. 5                       4.                       5. None of these
         5                 2                              2                        55

70.       Let {an} be strictly increasing sequence of positive integers such that a2 = 2, amn = am an for m, n
      co prime. What is the value of a100=?
      1. 99               2. 100                3. 101               4. Indeterminable 5. None of these




                         www.zioneducationonline.com                                    9899758900, 9891638000
                             Leading institute for MBA/GMAT/GRE/BBA/BCA/Hotel Mgt/Law entrance exams
                    Centres at: North Campus, Pitampura, Rajinder Nagar, Noida, Greater Noida, Indirapuram(Ghaziabad)

More Related Content

What's hot

synonyms and-antonyms
synonyms and-antonymssynonyms and-antonyms
synonyms and-antonyms
bilal8343
 
Naked Reading Ccisd
Naked Reading CcisdNaked Reading Ccisd
Naked Reading Ccisd
Teri Lesesne
 
ABI Dropbox: gp assignment IT.docx http://db.tt/cv4M99vt
ABI Dropbox: gp assignment IT.docx http://db.tt/cv4M99vtABI Dropbox: gp assignment IT.docx http://db.tt/cv4M99vt
ABI Dropbox: gp assignment IT.docx http://db.tt/cv4M99vt
bharathisenthil
 
ABI Dropbox: gp assignment IT.docx http://db.tt/cv4M99vt
ABI Dropbox: gp assignment IT.docx http://db.tt/cv4M99vtABI Dropbox: gp assignment IT.docx http://db.tt/cv4M99vt
ABI Dropbox: gp assignment IT.docx http://db.tt/cv4M99vt
bharathisenthil
 
Thi thu-anh-chuyen-nguyen-hue-2013
Thi thu-anh-chuyen-nguyen-hue-2013Thi thu-anh-chuyen-nguyen-hue-2013
Thi thu-anh-chuyen-nguyen-hue-2013
Bồ Anh
 

What's hot (20)

Let General Education reviewer
Let General Education reviewerLet General Education reviewer
Let General Education reviewer
 
Gen edu set b
Gen edu   set bGen edu   set b
Gen edu set b
 
Bo de-thi-thu-va-dap-an-thpt-quoc-gia-mon-tieng-anh-truong-thpt-hong-linh-nam...
Bo de-thi-thu-va-dap-an-thpt-quoc-gia-mon-tieng-anh-truong-thpt-hong-linh-nam...Bo de-thi-thu-va-dap-an-thpt-quoc-gia-mon-tieng-anh-truong-thpt-hong-linh-nam...
Bo de-thi-thu-va-dap-an-thpt-quoc-gia-mon-tieng-anh-truong-thpt-hong-linh-nam...
 
LET REVIEWER
LET REVIEWERLET REVIEWER
LET REVIEWER
 
General Education reviewer in passing the LET or BLEPT Examination
General Education reviewer in passing the LET or BLEPT ExaminationGeneral Education reviewer in passing the LET or BLEPT Examination
General Education reviewer in passing the LET or BLEPT Examination
 
City guilds c1 pr-test-1 sb
City guilds c1 pr-test-1 sbCity guilds c1 pr-test-1 sb
City guilds c1 pr-test-1 sb
 
500 Analogy practice questions by studyIQ coaching center
500 Analogy practice questions by studyIQ coaching center500 Analogy practice questions by studyIQ coaching center
500 Analogy practice questions by studyIQ coaching center
 
synonyms and-antonyms
synonyms and-antonymssynonyms and-antonyms
synonyms and-antonyms
 
Naked Reading Ccisd
Naked Reading CcisdNaked Reading Ccisd
Naked Reading Ccisd
 
De thi thu tieng anh co dap an
De thi thu tieng anh co dap anDe thi thu tieng anh co dap an
De thi thu tieng anh co dap an
 
ABI Dropbox: gp assignment IT.docx http://db.tt/cv4M99vt
ABI Dropbox: gp assignment IT.docx http://db.tt/cv4M99vtABI Dropbox: gp assignment IT.docx http://db.tt/cv4M99vt
ABI Dropbox: gp assignment IT.docx http://db.tt/cv4M99vt
 
ABI Dropbox: gp assignment IT.docx http://db.tt/cv4M99vt
ABI Dropbox: gp assignment IT.docx http://db.tt/cv4M99vtABI Dropbox: gp assignment IT.docx http://db.tt/cv4M99vt
ABI Dropbox: gp assignment IT.docx http://db.tt/cv4M99vt
 
LET reviewer in general education
LET reviewer in general educationLET reviewer in general education
LET reviewer in general education
 
Thi thu-anh-chuyen-nguyen-hue-2013
Thi thu-anh-chuyen-nguyen-hue-2013Thi thu-anh-chuyen-nguyen-hue-2013
Thi thu-anh-chuyen-nguyen-hue-2013
 
De thi-thu-thpt-quoc-gia-mon-tieng-anh-lan2-nam2015-truong-thpt-nong-cong1-th...
De thi-thu-thpt-quoc-gia-mon-tieng-anh-lan2-nam2015-truong-thpt-nong-cong1-th...De thi-thu-thpt-quoc-gia-mon-tieng-anh-lan2-nam2015-truong-thpt-nong-cong1-th...
De thi-thu-thpt-quoc-gia-mon-tieng-anh-lan2-nam2015-truong-thpt-nong-cong1-th...
 
The Dating Game
The Dating GameThe Dating Game
The Dating Game
 
Summary of the Book Made to Stick
Summary of the Book Made to Stick Summary of the Book Made to Stick
Summary of the Book Made to Stick
 
13- LET Filipino part 1
13- LET Filipino part 113- LET Filipino part 1
13- LET Filipino part 1
 
LET Sample Exam General education set b
LET Sample Exam General education set bLET Sample Exam General education set b
LET Sample Exam General education set b
 
Joe Ditzel Has Some Golf Problems
Joe Ditzel Has Some Golf ProblemsJoe Ditzel Has Some Golf Problems
Joe Ditzel Has Some Golf Problems
 

Viewers also liked

Tizen web app について調べたよ
Tizen web app について調べたよTizen web app について調べたよ
Tizen web app について調べたよ
Naruto TAKAHASHI
 
ITI Solutions - Comparative Information
ITI Solutions - Comparative InformationITI Solutions - Comparative Information
ITI Solutions - Comparative Information
Ben Lucas
 
How to use millennium create lists2
How to use millennium create lists2How to use millennium create lists2
How to use millennium create lists2
jodiesaw
 
European financial centres will survive the crisis’ – WELLSPHERE
European financial centres will survive the crisis’ – WELLSPHEREEuropean financial centres will survive the crisis’ – WELLSPHERE
European financial centres will survive the crisis’ – WELLSPHERE
alduckhum
 

Viewers also liked (20)

Get a Seat at the Table: Utah's Legislature and How it Impacts Nonprofit Orga...
Get a Seat at the Table: Utah's Legislature and How it Impacts Nonprofit Orga...Get a Seat at the Table: Utah's Legislature and How it Impacts Nonprofit Orga...
Get a Seat at the Table: Utah's Legislature and How it Impacts Nonprofit Orga...
 
Tizen & Crosswalk
Tizen & CrosswalkTizen & Crosswalk
Tizen & Crosswalk
 
Freello #Expo2015 #Milano
Freello #Expo2015 #MilanoFreello #Expo2015 #Milano
Freello #Expo2015 #Milano
 
Sa
SaSa
Sa
 
Tizen web app について調べたよ
Tizen web app について調べたよTizen web app について調べたよ
Tizen web app について調べたよ
 
ITI Solutions - Comparative Information
ITI Solutions - Comparative InformationITI Solutions - Comparative Information
ITI Solutions - Comparative Information
 
How to use millennium create lists2
How to use millennium create lists2How to use millennium create lists2
How to use millennium create lists2
 
Freello Eventi Live
Freello Eventi LiveFreello Eventi Live
Freello Eventi Live
 
August2012
August2012August2012
August2012
 
February 2015 UK Commercial Bulletin
February 2015 UK Commercial BulletinFebruary 2015 UK Commercial Bulletin
February 2015 UK Commercial Bulletin
 
Los 7 juegos de pat parelli bo
Los 7 juegos de pat parelli boLos 7 juegos de pat parelli bo
Los 7 juegos de pat parelli bo
 
golf clinic pacchetto vacanza hotel gianna madonna di campiglio
 golf clinic pacchetto vacanza hotel gianna madonna di campiglio golf clinic pacchetto vacanza hotel gianna madonna di campiglio
golf clinic pacchetto vacanza hotel gianna madonna di campiglio
 
Ebm
EbmEbm
Ebm
 
BIMobject® seminar 7th of may 2013 Denmark
BIMobject® seminar 7th of may 2013 DenmarkBIMobject® seminar 7th of may 2013 Denmark
BIMobject® seminar 7th of may 2013 Denmark
 
камянець подільський
камянець   подільськийкамянець   подільський
камянець подільський
 
Code4lib2013.09.01 fujiwara
Code4lib2013.09.01 fujiwaraCode4lib2013.09.01 fujiwara
Code4lib2013.09.01 fujiwara
 
Los 7 juegos de pat parelli
Los 7 juegos de pat parelliLos 7 juegos de pat parelli
Los 7 juegos de pat parelli
 
The Ins and Outs of the R15 Grant
The Ins and Outs of the R15 GrantThe Ins and Outs of the R15 Grant
The Ins and Outs of the R15 Grant
 
European financial centres will survive the crisis’ – WELLSPHERE
European financial centres will survive the crisis’ – WELLSPHEREEuropean financial centres will survive the crisis’ – WELLSPHERE
European financial centres will survive the crisis’ – WELLSPHERE
 
Maquina virtual
Maquina virtualMaquina virtual
Maquina virtual
 

Similar to Nat cat 10 full

(1) After Sean was arrested for breaking into a pawnshop, I began .docx
(1) After Sean was arrested for breaking into a pawnshop, I began .docx(1) After Sean was arrested for breaking into a pawnshop, I began .docx
(1) After Sean was arrested for breaking into a pawnshop, I began .docx
tienmixon
 
Student ID 21722970 Exam 250483RR - The Reading and Writing Proc.docx
Student ID 21722970 Exam 250483RR - The Reading and Writing Proc.docxStudent ID 21722970 Exam 250483RR - The Reading and Writing Proc.docx
Student ID 21722970 Exam 250483RR - The Reading and Writing Proc.docx
simba35
 

Similar to Nat cat 10 full (20)

How To Teach Essay Writing To High School Students - Tip
How To Teach Essay Writing To High School Students - TipHow To Teach Essay Writing To High School Students - Tip
How To Teach Essay Writing To High School Students - Tip
 
(1) After Sean was arrested for breaking into a pawnshop, I began .docx
(1) After Sean was arrested for breaking into a pawnshop, I began .docx(1) After Sean was arrested for breaking into a pawnshop, I began .docx
(1) After Sean was arrested for breaking into a pawnshop, I began .docx
 
My Last Day At School Essay In Urdu
My Last Day At School Essay In UrduMy Last Day At School Essay In Urdu
My Last Day At School Essay In Urdu
 
How To Make A 5 Paragraph Persuasive Essay
How To Make A 5 Paragraph Persuasive EssayHow To Make A 5 Paragraph Persuasive Essay
How To Make A 5 Paragraph Persuasive Essay
 
Essay On Cheating In Examination Hall
Essay On Cheating In Examination HallEssay On Cheating In Examination Hall
Essay On Cheating In Examination Hall
 
Korean sat (english) 2015
Korean sat (english) 2015Korean sat (english) 2015
Korean sat (english) 2015
 
Essay On English As A Global Language
Essay On English As A Global LanguageEssay On English As A Global Language
Essay On English As A Global Language
 
Sample Of Essay Writing For Toefl. Online assignment writing service.
Sample Of Essay Writing For Toefl. Online assignment writing service.Sample Of Essay Writing For Toefl. Online assignment writing service.
Sample Of Essay Writing For Toefl. Online assignment writing service.
 
Different Essay Types Ielts
Different Essay Types IeltsDifferent Essay Types Ielts
Different Essay Types Ielts
 
9 dpa am_zad_2015
9 dpa am_zad_20159 dpa am_zad_2015
9 dpa am_zad_2015
 
Write Critical Essay Example
Write Critical Essay ExampleWrite Critical Essay Example
Write Critical Essay Example
 
Ged Sample Essay Pdf
Ged Sample Essay PdfGed Sample Essay Pdf
Ged Sample Essay Pdf
 
Thesis Write Up Format - Thesis Format Requirement
Thesis Write Up Format - Thesis Format RequirementThesis Write Up Format - Thesis Format Requirement
Thesis Write Up Format - Thesis Format Requirement
 
Thesis Write Up Format - Thesis Format Requirement
Thesis Write Up Format - Thesis Format RequirementThesis Write Up Format - Thesis Format Requirement
Thesis Write Up Format - Thesis Format Requirement
 
Best Essays For Scholarships. Online assignment writing service.
Best Essays For Scholarships. Online assignment writing service.Best Essays For Scholarships. Online assignment writing service.
Best Essays For Scholarships. Online assignment writing service.
 
Student ID 21722970 Exam 250483RR - The Reading and Writing Proc.docx
Student ID 21722970 Exam 250483RR - The Reading and Writing Proc.docxStudent ID 21722970 Exam 250483RR - The Reading and Writing Proc.docx
Student ID 21722970 Exam 250483RR - The Reading and Writing Proc.docx
 
Act Essay Prompts 2011
Act Essay Prompts 2011Act Essay Prompts 2011
Act Essay Prompts 2011
 
7 Ways To Improve Your Essay Writing Skills
7 Ways To Improve Your Essay Writing Skills7 Ways To Improve Your Essay Writing Skills
7 Ways To Improve Your Essay Writing Skills
 
Practice Writing An Essay Online. Online assignment writing service.
Practice Writing An Essay Online. Online assignment writing service.Practice Writing An Essay Online. Online assignment writing service.
Practice Writing An Essay Online. Online assignment writing service.
 
Essay For Merit Scholarship. Online assignment writing service.
Essay For Merit Scholarship. Online assignment writing service.Essay For Merit Scholarship. Online assignment writing service.
Essay For Merit Scholarship. Online assignment writing service.
 

Recently uploaded

Making and Justifying Mathematical Decisions.pdf
Making and Justifying Mathematical Decisions.pdfMaking and Justifying Mathematical Decisions.pdf
Making and Justifying Mathematical Decisions.pdf
Chris Hunter
 
1029-Danh muc Sach Giao Khoa khoi 6.pdf
1029-Danh muc Sach Giao Khoa khoi  6.pdf1029-Danh muc Sach Giao Khoa khoi  6.pdf
1029-Danh muc Sach Giao Khoa khoi 6.pdf
QucHHunhnh
 
Beyond the EU: DORA and NIS 2 Directive's Global Impact
Beyond the EU: DORA and NIS 2 Directive's Global ImpactBeyond the EU: DORA and NIS 2 Directive's Global Impact
Beyond the EU: DORA and NIS 2 Directive's Global Impact
PECB
 

Recently uploaded (20)

Explore beautiful and ugly buildings. Mathematics helps us create beautiful d...
Explore beautiful and ugly buildings. Mathematics helps us create beautiful d...Explore beautiful and ugly buildings. Mathematics helps us create beautiful d...
Explore beautiful and ugly buildings. Mathematics helps us create beautiful d...
 
Mattingly "AI & Prompt Design: The Basics of Prompt Design"
Mattingly "AI & Prompt Design: The Basics of Prompt Design"Mattingly "AI & Prompt Design: The Basics of Prompt Design"
Mattingly "AI & Prompt Design: The Basics of Prompt Design"
 
INDIA QUIZ 2024 RLAC DELHI UNIVERSITY.pptx
INDIA QUIZ 2024 RLAC DELHI UNIVERSITY.pptxINDIA QUIZ 2024 RLAC DELHI UNIVERSITY.pptx
INDIA QUIZ 2024 RLAC DELHI UNIVERSITY.pptx
 
Mattingly "AI & Prompt Design: Structured Data, Assistants, & RAG"
Mattingly "AI & Prompt Design: Structured Data, Assistants, & RAG"Mattingly "AI & Prompt Design: Structured Data, Assistants, & RAG"
Mattingly "AI & Prompt Design: Structured Data, Assistants, & RAG"
 
Key note speaker Neum_Admir Softic_ENG.pdf
Key note speaker Neum_Admir Softic_ENG.pdfKey note speaker Neum_Admir Softic_ENG.pdf
Key note speaker Neum_Admir Softic_ENG.pdf
 
fourth grading exam for kindergarten in writing
fourth grading exam for kindergarten in writingfourth grading exam for kindergarten in writing
fourth grading exam for kindergarten in writing
 
Unit-V; Pricing (Pharma Marketing Management).pptx
Unit-V; Pricing (Pharma Marketing Management).pptxUnit-V; Pricing (Pharma Marketing Management).pptx
Unit-V; Pricing (Pharma Marketing Management).pptx
 
Measures of Dispersion and Variability: Range, QD, AD and SD
Measures of Dispersion and Variability: Range, QD, AD and SDMeasures of Dispersion and Variability: Range, QD, AD and SD
Measures of Dispersion and Variability: Range, QD, AD and SD
 
Mehran University Newsletter Vol-X, Issue-I, 2024
Mehran University Newsletter Vol-X, Issue-I, 2024Mehran University Newsletter Vol-X, Issue-I, 2024
Mehran University Newsletter Vol-X, Issue-I, 2024
 
Making and Justifying Mathematical Decisions.pdf
Making and Justifying Mathematical Decisions.pdfMaking and Justifying Mathematical Decisions.pdf
Making and Justifying Mathematical Decisions.pdf
 
APM Welcome, APM North West Network Conference, Synergies Across Sectors
APM Welcome, APM North West Network Conference, Synergies Across SectorsAPM Welcome, APM North West Network Conference, Synergies Across Sectors
APM Welcome, APM North West Network Conference, Synergies Across Sectors
 
Advanced Views - Calendar View in Odoo 17
Advanced Views - Calendar View in Odoo 17Advanced Views - Calendar View in Odoo 17
Advanced Views - Calendar View in Odoo 17
 
Código Creativo y Arte de Software | Unidad 1
Código Creativo y Arte de Software | Unidad 1Código Creativo y Arte de Software | Unidad 1
Código Creativo y Arte de Software | Unidad 1
 
1029-Danh muc Sach Giao Khoa khoi 6.pdf
1029-Danh muc Sach Giao Khoa khoi  6.pdf1029-Danh muc Sach Giao Khoa khoi  6.pdf
1029-Danh muc Sach Giao Khoa khoi 6.pdf
 
How to Give a Domain for a Field in Odoo 17
How to Give a Domain for a Field in Odoo 17How to Give a Domain for a Field in Odoo 17
How to Give a Domain for a Field in Odoo 17
 
Paris 2024 Olympic Geographies - an activity
Paris 2024 Olympic Geographies - an activityParis 2024 Olympic Geographies - an activity
Paris 2024 Olympic Geographies - an activity
 
Web & Social Media Analytics Previous Year Question Paper.pdf
Web & Social Media Analytics Previous Year Question Paper.pdfWeb & Social Media Analytics Previous Year Question Paper.pdf
Web & Social Media Analytics Previous Year Question Paper.pdf
 
Beyond the EU: DORA and NIS 2 Directive's Global Impact
Beyond the EU: DORA and NIS 2 Directive's Global ImpactBeyond the EU: DORA and NIS 2 Directive's Global Impact
Beyond the EU: DORA and NIS 2 Directive's Global Impact
 
ICT Role in 21st Century Education & its Challenges.pptx
ICT Role in 21st Century Education & its Challenges.pptxICT Role in 21st Century Education & its Challenges.pptx
ICT Role in 21st Century Education & its Challenges.pptx
 
psychiatric nursing HISTORY COLLECTION .docx
psychiatric  nursing HISTORY  COLLECTION  .docxpsychiatric  nursing HISTORY  COLLECTION  .docx
psychiatric nursing HISTORY COLLECTION .docx
 

Nat cat 10 full

  • 1. MOCK CAT VERBAL ABILITY AND READING COMPREHENSION DIRECTIONS for Questions 1 & 2: Read the arguments and answer the questions that follow. 1. Housing property rentals in city A should go up even more. This will attract more landlords to rent out their houses, which in turn will improve the availability of houses in city A. Thus, bringing down the housing property rentals in the long run. Of the following, which is the assumption made in the above paragraph? (1) Rents are low at present. (2) Landlords are interested only in rents. (3) Tenants are willing to pay more. (4) In the long run, the number of houses available for rent will exceed the number of people looking for houses. (5) Managing property rentals is an organized activity in city A. 2. Children who consistently learn music or painting at an early age demonstrate a deeper understanding of their environment and are able to cope with their peers and seniors more amicably. Due to this, educators believe that getting children to learn music or painting at an early age will increase and hone their social skills. Which of the following statements shows a flaw in the reasoning presented above? (1) A child’s social skills usually develop at an early stage and remain fixed into his personality. (2) Development of social skills depends on the exposure that a child receives through interacting and conversing with his parents, siblings, relatives and friends. (3) Social skills are essential for any child as they are also responsible for shaping the personality of the child. (4) Peers and seniors can also help students who feel shy or uncomfortable while interacting with other at school or outside of it. (5) Children, who possess good social skills go for learning music and painting. DIRECTIONS for Questions 3 & 4: Each of the following questions has a paragraph from which the first sentence has been deleted. From the given options choose the one that begins the paragraph in the most appropriate way. 3. It was the impact of personal events that forced me to translate everything on paper. It was anger, jealousy and bitterness that shaped my narration and sketched extremely complex characters. I shared my pain with them. As my real life situation became more complex, the suffering and www.zioneducationonline.com 9899758900, 9891638000 Leading institute for MBA/GMAT/GRE/BBA/BCA/Hotel Mgt/Law entrance exams Centres at: North Campus, Pitampura, Rajinder Nagar, Noida, Greater Noida, Indirapuram(Ghaziabad)
  • 2. bitterness of my characters increased. Some of them became real monsters, while others showed fewer signs of sanity. I was amazed at the powerful creations that my emotions had unleashed. Not only had my emotions helped to create a world of my own; they had made me a writer as well. (1) Earlier when I wrote, I undermined the power of emotions and preferred to be inspired by facts. (2) When I was disgusted or hurt, I wrote. (3) Writing style truly represents the source of a writer’s inspiration and ideas. (4) My negative experiences translated into vicious characters on paper. (5) I am a victim of my emotions. 4. Owned by the Wrigleys, this magnificent piece of art had walls made from Portland stone. The open lobby was about 40 feet by 32 feet with four pillars that supported the first floor. Fifteen bedrooms containing unique mahogany wood work, five splendid dining areas and a picturesque lawn adorned this property. It was an unknown architect’s brilliant creation that displayed a refined taste and superior technical knowledge. (1) For centuries, the Wrigley family had bought numerous properties all over the world. (2) The Wrigley’s mansion was one of the best properties in this area. (3) The mansion was not too different from any others in this part of the country. (4) Its architectural beauty had drawn many admirers from all over the world. (5) The house was , if it could be called one, was magnificient. DIRECTIONS for Questions 5 to 7: Read the passage given below and answer the questions that follow. “A boy is not happy owing to his age; boys who are called happy are being congratulated by reason of the hopes we have for them. For there is required not only complete virtue, but also a complete life, since many changes occur in life, and all manner of chances, and the most prosperous may fall into great misfortunes in old age.” In other words, what Aristotle is saying is that what is required for happiness is “a complete life” which obviously no young person has while he is still young. He makes the same point in another way. He refers to the story of Croesus and Solon, as told by the ancient Greek historian, Herodotus. Croesus was King of Lydia, and one of the richest and most powerful rulers of his day. Solon was one of the wisest men of Greece. Here is the story of their conversation. “Solon set out upon his travels, in the course of which he came on a visit to Croesus at Sardis. Croesus received him as his guest, and lodged him in the royal palace, and had his servants conduct him over his treasures, and show him all their greatness and magnificence. And when Solon had seen them all. Croesus said, ‘Stranger of Athens, I have heard much of your wisdom and of your travels through many lands. I am curious therefore to ask you, whom of all the men that you have seen, you consider the most happy?’ This he asked because he thought himself the happiest of mortals: but Solon answered him without flattery: ‘Tell us of Athens, sire.’ Astonished at what he heard, Croesus demanded sharply, ‘And why do you consider Tell us the happiest of men?’ To which the other replied, ‘First because his country was flourishing in his days, and he himself had sons both beautiful and good, and he lived to see children born to each of them, and these children all grew up; and further because, after a life spent in what our people look upon as comfort his end was glorious. In a battle between tie Athenians and their neighbors near Eleusis, he died gallantly upon the field. And the Athenians gave him a public funeral and paid him the highest honors.” Thus, Solon admonished Croesus by the example of Tell us. www.zioneducationonline.com 9899758900, 9891638000 Leading institute for MBA/GMAT/GRE/BBA/BCA/Hotel Mgt/Law entrance exams Centres at: North Campus, Pitampura, Rajinder Nagar, Noida, Greater Noida, Indirapuram(Ghaziabad)
  • 3. When he had ended, Croesus asked angrily, ‘Is my happiness, then, so little to you that you do not even put me on a level with private men?’ 5. It can be inferred from the passage that according to Aristotle (1) Young people cannot be happy. (2) Young people lack complete virtue. (3) Young people can be happy if they have children. (4) There are many factors which need to be fulfilled for a young person to be happy. (5) Young people can be happy if they have high hopes. 6. Which of the following is most likely to follow the passage as Solon’s answer to Croesus? (1) ‘I see that You are wonderfully rich and are the lord of many nations, and as for your question, I would like to hear of the wealth that you have amassed’. (2) ‘I see that You are wonderfully rich and are the lord of many nations, but as for your question, I have no answer to give until I hear that you have closed your life happily’ (3) ‘I see that You are wonderfully rich and are the lord of many nations, but as for your question, I have no answer to give until I hear about your children and grandchildren’ (4) ‘I see that You are wonderfully rich and are the lord of many nations, but as for your question, I have no answer to give until I hear that you have led a virtuous life’. (5) ‘I see that You are wonderfully rich and are the lord of many nations, but as for your question, I have no answer to give until I hear how you have faced misfortune’. 7. A tone /emotion which is not part of the passage would be? (1) Rebuke (2) Rhetorical (3) Narration 4) Didactic (5) Pity DIRECTIONS for Questions 8 to 10: Arrange the sentences A, B, C and D to form a logical sequence between sentences 1 and 6. 8. 1. Meeting people after nine to ten years, almost to the day, is a very weird experience. A. It genuinely felt awkward to meet people, some married and some with children, others married but who forgot to send out 200 cards to school friends, others, divorced. B. I did find out that the marriage was unhappy for all the wrong reasons, none because of the obnoxious twit that he is. C. I actually felt sorry for one of those guys, because, and if you knew equation with him in the school bus where we almost killed each other a few times, I would not have wished a divorce on him -though, I would not have wished any woman on him either. D. I am in close touch with a couple of school friends - Doc, for example is an ass I can still call my best friend after twenty years - but my god, did he (or rather his overheating BMW) push my patience on Saturday night. 6. Everybody was fatter/ balder and in some cases both. (1) DCBA (2) CBDA (3) DACB (4) BCDA (5) ABCD 9. 1. Even though dance starts a bit later than school does during the week, Saturday morning is still pretty chaotic around here. www.zioneducationonline.com 9899758900, 9891638000 Leading institute for MBA/GMAT/GRE/BBA/BCA/Hotel Mgt/Law entrance exams Centres at: North Campus, Pitampura, Rajinder Nagar, Noida, Greater Noida, Indirapuram(Ghaziabad)
  • 4. A. More so if John is leaving for an auction that day, because it means I have to truck Kristen and Alex with me and get them ready too. B. This is one of those mornings where John was rushing to leave too. C. There are buns to be done, bodysuits to find, tights to mend (because they’re always ripped somewhere) and a good breakfast to be had. D. His work van has been giving him some problems, so he was nervous about travelling with it (not to mention what it’s costing us to fix it). 6. He couldn’t find his cell and even though he had woken up in a general good mood, I could see it going downhill from there. (1) ACBD (2) ABCD (3) ADCB (4) BCDA (5) BDCA 10. 1. Climate change will trigger a chain of events that is likely to prompt an increase in HIV rates world wide, an expert has warned. A. Daniel Tarantola of the University of New South Wales (UNSW) said the disadvantage in developing countries must be addressed if the world is to prevent a dramatic escalation of the HIV epidemic as well as other health problems. B. "It was clear soon after the emergence of the HIV epidemic that discrimination, gender inequality and lack of access to essential services have made some populations more vulnerable than others, "Tarantola said on Wednesday. C. "Today, additional threats are lurking on the horizon as the global economic situation deteriorates, food scarcity worsens and climate change begins to affect those who were already dependent on survival economies," Tarantola said. D. David Cooper, also of UNSW, said: “Science has achieved great strides towards shaping a more effective response to HIV. 6. Yet research has not succeeded in producing the hoped-for 'magic bullets' of either a cure or a vaccine.” (1) BACD (2) DCBA (3) CDBA (4) ABCD (5) ADCB DIRECTIONS for questions 11 to 14: Fill in the blanks with the correct option given below: 11. Rich in local color, the undercurrents that run beneath the seemingly ………… surroundings of the sleepy village are explored in fluid prose. (1) pristine (2) idyllic (3) clarity (4) liquid (5) lazy 12. With models and beauty queens becoming ……..for the youth, fashion is a …… sector. (1) icons; thriving (2) idols; recommended (3) ideals; booming (4) None of these (5) mentors; slowing 13. Leaving the country of one's ……… for a new land is a major decision but globalization has taken away the fear and lessened the anxiety. (1) growth (2) death (3) birth (4) progress (5) family www.zioneducationonline.com 9899758900, 9891638000 Leading institute for MBA/GMAT/GRE/BBA/BCA/Hotel Mgt/Law entrance exams Centres at: North Campus, Pitampura, Rajinder Nagar, Noida, Greater Noida, Indirapuram(Ghaziabad)
  • 5. 14. As the second generation of the post 1965 Indian immigrants come of age in America, we are beginning to see a wealth of literature as well as critical essays in which they …………… and …………. their experiences. (1) chronicle; analyse (2) extend; elaborate (3) list; expunge (4) sort; practice (5) decry ;dissuade DIRECTIONS for questions 15 to 17: Read the passage and answer whether the statements that follow are: (A) a logical conclusion to the passage. (B) a contradiction to the intent of the passage. (C) a far-fetched conclusion. (D) Irrelevant to the passage. Passage Ramesh Melwani joined a leading leather company. An entrepreneur-driven, fast growing, exporter of leather accessories, the company had been doubling its turnover every year for the last six years. Melwani joined with a 2.5-fold jump in salary. Within a week he was back in his old company. Reason: he couldn't survive the ruthless, 16-hour-a-day regimen, where the individual was given no respect. Today he's a lot poorer, a lot happier. 15. High salaries do not necessarily motivate or help retain talent in an organisation. 16. The entrepreneur alone can make or break an organisation. 17. The irrevocable bottom line in the new age organisations is human resource development. DIRECTIONS for question 18 Relate to the following passage and answer the question: An important question remains: Why has atheism and skepticism toward religion suddenly emerged as a question of great current interest, at least among the literate, in the past few years? Clearly something has happened to break atheists of their tendency to nurture their disbelief privately and to keep their opinions to themselves. It seems obvious that politics has a lot to do with this, specifically the cultivation of the religious right as a phalanx of conservative storm-troopers who are rewarded by conservative politicians by having their singular dogmas incorporated, as much as possible, into public policy. The increasing pressure on women’s reproductive rights, the suppression of stem cell research and, most egregiously, the fresh intrusion of creationism into public schools are primary instances of this. Beyond these concrete horrors, there is no escaping the fact that the miasma of compulsory religiosity has thickened and diffused throughout society. For instance, one notes, rather queasily, the success of the Evangelicals in turning the Air Force Academy into a virtual fundamentalist seminary where cadets from all sorts of backgrounds are relentlessly pressured by officers and upperclassmen into declaring for the Born- Agains. www.zioneducationonline.com 9899758900, 9891638000 Leading institute for MBA/GMAT/GRE/BBA/BCA/Hotel Mgt/Law entrance exams Centres at: North Campus, Pitampura, Rajinder Nagar, Noida, Greater Noida, Indirapuram(Ghaziabad)
  • 6. Atheists, who, despite polls, have never been all that rare, have come to mistrust the notion that they can believe as they will, undisturbed, provided they remain discreet about it. The mood fostered by the religious right seems to tend toward the inquisitorial. Scientists, in particular, representing the one vocation in which non-belief is the norm, rather than the outlier, have sensibly concluded that the culture in which they have quietly lived is being attacked at its foundations. It’s one thing to send your kids to a public school where “under god” is formulaically recited as part of the daily Pledge of Allegiance. It’s quite another to have your kid’s elementary biology class interrupted by harangues against ‘Darwinism’, or to see the Bible, taught as literal truth, surreptitiously introduced into the curriculum. When matter shave come to that pass, scientists, among others, see little point in not fighting back openly. Thus, one now sees a torrent of books, largely by scientists and sympathetic philosophers, striking back not only at the enemies of stem-cell research and the proponents of Intelligent Design Theory, but at the very roots of the cultural tic that provides these miscreants such fertile ground: supernatural religion predicated upon a supreme being. Then, too, one must consider the possibility that, as the ostensible religious uniformity of the nation begins to dissolve, the sort of person who habitually responds to questionnaires about faith by listing himself as ‘unaffiliated, but a believer’ has, at long last, become more honest with himself and with the rest of us, frankly acknowledging the deep vein of skepticism underlying his disinclination to become a churchgoer. I suspect that many of the readers of the New Atheists’ books fall into this category, and are more willing to let their doubts see the light of day. 18. In the light of your reading of the passage above, identify the option that contains the set of words CLOSEST in the meaning to the words in CAPITALS: EGREGIOUS: SKEPTICISM: INQUISITORIAL (A) blatant: suspicion : intolerant (B) intolerant : certitude : keen (C) balanced: confident : inane (D) mean : interrogation : impatient (E) unsure: bored: crazy DIRECTIONS for Question 19: Each of the following questions has a paragraph from which the last sentence has been deleted. From the given options, choose the one that completes the paragraph in the most appropriate way. 19. The difference between the revolutionary and the terrorist lies in the reason for which each fights. For whoever stands by a just cause and fights for the freedom and liberation of his land from the invaders, the settlers and the colonialists cannot possibly be called terrorist, otherwise the American people in their struggle for liberation from the British colonialists would have been terrorists the European resistance against the Nazis would be terrorism, the struggle of the Asian, African and Latin American peoples would also be terrorism, and many of you who are in this Assembly hall were considered terrorists. This is actually a just and proper struggle consecrated by the United Nations Charter and by the Universal Declaration of Human Rights. As to those who fight against the just causes, those who wage war to occupy, colonize and oppress other people, those are the terrorists. (1) They are the ones who invade everywhere; they are the ones with no clear goal. (2) Because the justice of the cause determines the right to struggle. (3) The end is as important as the means. www.zioneducationonline.com 9899758900, 9891638000 Leading institute for MBA/GMAT/GRE/BBA/BCA/Hotel Mgt/Law entrance exams Centres at: North Campus, Pitampura, Rajinder Nagar, Noida, Greater Noida, Indirapuram(Ghaziabad)
  • 7. (4) I urge everyone present here to resound this with me that the people of Palestine are not terrorist s. (5) Not the Asians, not the Africans and not the Latin Americans for sure. DIRECTIONS for Question 20 to 22: Each of the following passages are followed by an inference or inferences. Examine each inference separately in the context of the passage. Each inference is denoted by: 1 if the inference is definitely true, i.e. it directly follows from the statement of facts given. 2 if the inference is probably true, though not directly true, in the light of the statement of facts given. 3 if the inference is uncertain, i.e. data is insufficient to decide whether the inference is true or false. 4 if the inference is probably false, though not definitely false, in the light of the statement of facts given. 5 if the inference is definitely false, i.e. it cannot possibly be inferred from the statement of facts given. Choose the option that gives the most appropriate set of conclusions about the inference or inferences. 20. Native Chinese geography begins in the Warring States period (5th century BC). It expands its scope beyond the Chinese homeland with the growth of the Chinese Empire under the Han Dynasty. It enters its golden age with the invention of the compass in the 11th century (Song Dynasty) and peaks with 15th century (Ming Dynasty) Chinese exploration of the Pacific under admiral Zheng He. Inference: China was ruled by the Han dynasty at some point of time in its history. (1) Option 1 (2) Option 2 (3) Option 3 (4) Option 4 (5) Option 5 21. Pop music is an ample and imprecise category of modern music not defined by artistic considerations but by its potential audience or prospective market. Pop is music composed with deliberate intent to appeal to the majority of its contemporaries. Inference: Pop music is more commercial in nature than all other forms of music. (1) Option 1 (2) Option 2 (3) Option 3 (4) Option 4 (5) Option 5 22. A compass (or mariner compass) is a navigational instrument for finding directions on the Earth. It consists of a magnetized pointer free to align itself accurately with Earth's magnetic field, which is of great assistance in navigation. Inference: Most modern ships use a compass to navigate their way around oceans. (1) Option 1 (2) Option 2 (3) Option 3 (4) Option 4 (5) Option 5 DIRECTIONS for Question 23 to 25:Each of the questions below has a set of sequentially ordered statements. Each statement can be classified as one of the following: www.zioneducationonline.com 9899758900, 9891638000 Leading institute for MBA/GMAT/GRE/BBA/BCA/Hotel Mgt/Law entrance exams Centres at: North Campus, Pitampura, Rajinder Nagar, Noida, Greater Noida, Indirapuram(Ghaziabad)
  • 8. A. Facts, which deal with the pieces of information that one has heard, seen or read, and which are open to discovery or verification (the answer option indicates such a statement with an ‘F’) B. Inferences, which are conclusions drawn about the unknown, on the basis of the known (the answer option indicates such a statement with an ‘I’) C. Judgements, which are opinions that imply approval or disapproval of persons, objects, situations and occurrences in the past, the present or the future (the answer option indicates such a statement with a ‘J’) Select the answer option that best describes the set of statements. 23. A. Community is our stickiest glue, and nothing bonds us as willingly as our clan food. B. So, even if Bhajji now bats for Mumbai, nothing will ever make us stop rooting for the kind of roti we ate in our childhood kitchens. C. The handi that rocked our cradle will always rule our world. D. Cities also have a culinary DNA, and it is not easy to auction them off to the highest pizza franchise. E. It is worse for cities which stagger under the burden of their legendary cuisines. (1) IJIJI (2) JIJIJ (3) IIJJI (4) JJJJJ (5) JJIIJ 24. A. No one needs to be reminded of the ferment which is moving in the world of social affairs, of the obscure but powerful tendencies which are forcing society out of its grooves and leaving it, aspiring but dubious, in new and uncharted regions. B. This may affect different minds in different ways. C. Time- honoured political and economic formulae are become "old clothes" for an awakened and ardent generation, and before the new garments are quite ready the old one’s are shed. D. The blessed word "reconstruction" is often mentioned. (1) JIJF (2) JFFF (3) JFJI (4) IIJI (5) JIJI 25. A. It seems that men are not satisfied that society has really developed so successfully as it might have done; many believe that it finds itself in a cul-de-sac. B. The experienced can see that many of the offered reforms are but the repetition of old mistakes which will involve us in the same unhappy cycle of disillusion and failure. C. It is not to be wondered, if men everywhere are seeking for a sign, a glimpse of a scheme of life, a view of reality, a hint of human destiny and the true outcome of human effort, to be an inspiration and a guide to them in their pathetic struggle out of the morass. D. If Philosophy has anything to say which is to the point, then let Philosophy by all means say it. E. The people may indeed expect too much from it, as those who best grasp the measure of Philosophy's task are the first to urge an outcome. (1) IFJJF (2) FFIJJ (3) IFJJI (4) JFJJJ (5) FFJJJ DATA INTERPRETATION AND LOGICAL REASONING. www.zioneducationonline.com 9899758900, 9891638000 Leading institute for MBA/GMAT/GRE/BBA/BCA/Hotel Mgt/Law entrance exams Centres at: North Campus, Pitampura, Rajinder Nagar, Noida, Greater Noida, Indirapuram(Ghaziabad)
  • 9. 26. If it is true that ‘Neither sham is true nor it is practical’, then which one of the following statements is true? (1) Sham is true; however, it is not practical. (2) Sham is true and it is practical. (3) Sham is practical and untrue. (4) Sham is not practical and it is not true. (5) None of the above. 27. Identify the statement which is False, if it is true that ‘no people other than the members of the club are allowed inside the club premises’. (1) Some people are members of the club. (2) All members of the club may be allowed inside the club premises. (3) All people are allowed inside the club premises. (4) All those who are allowed inside the club premises are members of the club. (5) None of the above. DIRECTIONS for Questions 28 and 31: In this caselet whole numbers (not fractions of numbers) from 1 to 26 have been replaced by different letters of the alphabet. The sums below will enable you to crack the code. NB - The only arithmetical signs used in this puzzle are ‘+’ (plus), ‘–’ (minus) and ‘ × ’ (times). 1. A+B=C 9. R+J=S 2. D=E+F 10. S+T=U 3. F × F=G 11. W=N+Q 4. H=I+J 12. V+V=W 5. K=L+M 13. X ×Y = K 6. M × M=N 14. Y+B=Z 7. N+F=O 15. Z=P+S 8. P× Q = R 28. If 20 ≤ G ≤ 26 then G = ? 1. 21 2. 27 3. 25 4. 36 5. 40 29. If K = 24 and X is prime then Y = ? 1. 8 2. 6 3. 12 4. 24 5. 36 30. If S = 13 then R = ? 1.10 2.12 3. 9 4. 8 5. 6 31. If N is a square of a number which is the square of a prime number then N = ? 1. 81 2. 16 3. 625 4. 256 5. 125 DIRECTIONS for Questions 32 to 35: Answer the questions on the basis of the information given below. Devendra has 15 different locks viz. L1, L2 through L15 The locks were classified into three categories based upon their number of levers. L1, L15, L13 and L4 were of 8 levers each. L6, L2, L7, L9 and L14 were of6 levers each and the remaining locks were of 10 levers each. He selects five out of these fifteen www.zioneducationonline.com 9899758900, 9891638000 Leading institute for MBA/GMAT/GRE/BBA/BCA/Hotel Mgt/Law entrance exams Centres at: North Campus, Pitampura, Rajinder Nagar, Noida, Greater Noida, Indirapuram(Ghaziabad)
  • 10. locks on each of the eight consecutive days viz. Day 1, Day 2, Day 3, Day 4, Day 5, Day 6, Day 7 and Day 8. On each day he tries to unlock the selected locks but is successful in unlocking only one of the selected locks. If he unlocks any lock on a particular day, that lock is not selected by him on the subsequent days. It is also known that out of the locks unlocked by him, the number of locks of 6 levers is less than the number of locks of 8 levers. The following table gives details about the locks selected by him on the given six days. Day 1 L1 L7 L8 L4 L11 Day 2 L3 L15 L14 L12 L8 Day 3 L2 L7 L15 L9 L10 Day 4 L15 L13 L10 L3 L6 Day 5 L2 L15 L9 L10 L5 Day 6 L13 L6 L1 L8 L10 Day 7 L14 L11 L8 L2 L13 Day 8 L5 L6 L10 L14 L11 32. On how many days it is possible to uniquely determine which lock was unlocked by Devendra? (1) Three (2) Four (3) Five (4) Six (5) Seven 33. If after day 8 Devendra found that aggregate levers of all the locks that he has unlocked is minimum then the lever of the lock unlocked on day 7 is (1) 6 (2) 8 (3) 10 (4) Either (1) or (2) (5) Either (1) or (3) Additional Information for questions 34 & 35: After Day 8 Devendra found that aggregate levers of all the locks that he has unlocked is more than 64 but not more than 68. 34. Which of the following locks is definitely unlocked by Devendra? (1) L15 (2) L13 (3) L10 (4) L11 (5) Cannot be determined 35. If L9 is not unlocked and L8 is unlocked by Devendra, then which of the following can be the lock that is unlocked by him? (1) L2 (2) L5 (3) L11 (4) L14 (5) L10 DIRECTIONS for Questions 36 to 39 : Refer to the following data : In the Institute of Physical Education in Moskwa, grades are awarded according to the following arrangement : If percentage score is upto 10 – Grade I If percentage score is more than 10 and upto 25 – Grade H If percentage score is more than 25 and upto 37 – Grade G If percentage score is more than 37 and upto 50 – Grade F If percentage score is more than 50 and upto 60 – Grade E If percentage score is more than 60 and upto 70 – Grade D www.zioneducationonline.com 9899758900, 9891638000 Leading institute for MBA/GMAT/GRE/BBA/BCA/Hotel Mgt/Law entrance exams Centres at: North Campus, Pitampura, Rajinder Nagar, Noida, Greater Noida, Indirapuram(Ghaziabad)
  • 11. If percentage score is more than 70 and upto 80 – Grade C If percentage score is more than 80 and upto 90 – Grade B If percentage score is more than 90 and upto 100 – Grade A Following table gives raw scores for four students of the institute, stored in a computer. Scores of Evanova, however, have been lost due to an error. 1st Semester 2nd 3rd 4th Semester Semester Semester Anne 38 89 93 0 Bette 51 71 65 21 Cynthia 71 28 70 10 Debora 80 25 83 90 Evanova It is known that a grade each (out of the four) of Evanova is common with (same as) another student’s grade (though not necessarily of the same semester). However, one of Cynthia’s grades is the same as Evanova’s in the same semester. Bette’s two of the grades are same as Evanova’s but in different semesters. Similarly, Debora’s two of the grades are also same as Evanova’s but in different semesters. Anne has only one grade common with Evanova (that too in a different semester). Except as mentioned above, the students have no common grades. 36. What is the minimum marks that Evanova could have received in any one semester ? 1. 0 2. 11 3. 26 4. 38 5. 40 37. Evanova’s grades are same as Cynthia’s in which of the following semesters ? 1. 1st 2. 2nd 3. 3rd 4. 4th 5. None 38. If grade F and lower grades are termed ‘fail grades’, how many ‘fail grades’ were awarded to the five students during the four semesters ? 1. 6 2. 7 3. 8 4. 9 5. 10 39. If it is assumed for want of availability of actual marks that Evanova’s score for any semester is mid-value of the class corresponding to the grade, her cumulative score for the four semesters was nearest to which other student’s score ? 1. Anne 2. Bette 3. Cynthia 4. Debora 5. Anne & Bette both DIRECTIONS for 40 to 43: Answer each of these questions independently: 40. From a bag containing 100 balls, one ball weighs 9 grams and all the other weigh 10 grams each. Using a simple balance where balls can be kept on either pan, what is the minimum weighs required to identify the defective ball? (1) 3 (2) 4 (3) 5 (4) 7 (5) Cannot be determined www.zioneducationonline.com 9899758900, 9891638000 Leading institute for MBA/GMAT/GRE/BBA/BCA/Hotel Mgt/Law entrance exams Centres at: North Campus, Pitampura, Rajinder Nagar, Noida, Greater Noida, Indirapuram(Ghaziabad)
  • 12. 41. Blue ties go only with yellow or brown jackets. Red ties go only with brown or blue jackets. White shoes go only with blue ties or brown jackets. Which of the following is a valid dressing combination? (1) White shoes, Brown jackets and Blue ties. (2) Brown jacket, Red tie and White shoes. (3) White shoes, Blue jacket and Red ties. (4) Blue Jacket, White Tie, Red Shoes (5) All of the above. 42. 4 books were distributed among 3 friends, such that each received atleast 1 book. X received Ayn Rand’s Fountainhead. Y did not receive any Ayn Rand book. Z did not receive Stephen Covey’s 7 habits or Shiv Khera’s Born to win. If the fourth book was Ayn Rand’s Night of January 16th, who received the 7 habits? (1) X (2) Y (3) Z (4) Indeterminate (5) None 43. There are 100 statements written on the board. Statement 1 is “only 1 statement among the 100 is false”. Statement 2 is “ only 2 statements among the 100 are false”. Hence each nth statement reads “Only n statements among the 100 are false”. How many statements on the board are true if the last statement is “Only 100 statements among the 100 are false”? (1) 0 (2) 1 (3) 50 (4) Indeterminate (5) 99 DIRECTIONS for questions 44 & 45: Refer to the data below. Vibha has a choice of the following fruits for dinner - oranges, mangoes, bananas, pineapple, peaches, apples, guavas, and sweet lime. Given below is some additional information: Oranges and sweet lime are citrous fruits. Vibha has four fruits for dinner on any given day. Vibha does not have citrous fruits along with mangoes. Vibha eats peaches only if she eats sweet lime. Vibha has at least one of the three fruits - bananas, guavas, apples - everyday. Only one unit of any fruit can be eaten. 44. If Vibha eats peaches, which of the following fruits can she not eat on the same day? (1) Banana (2) Guava (3) Mango (4) Orange (5) Apples 45. Vibha eats sweet lime and guavas, then the number of possible combinations is (1) 10 (2) 14 (3) 20 (4) 16 (5) 24 PROBLEM SOLVING Q46. Bhuvanesh alone can build a wall in 15 days while Kalpesh can demolish the same by himself in 25 days. If Bhuvanesh starts working on odd date, Gurneet helps him. Ivneet helps Kalpesh only if Gurneet helps Bhuvanesh. In a day, Gurneet does half as much work as Kalpesh and Ivneet does half as much work as Bhuvanesh. Find the minimum number of days required to build the wall if Bhuvanesh abd Kalpesh work on alternate days? 1) 150 2) 75 3) 151 4) 201 5) None of these Q47. The hundred’s place digit of (251)^148 is 1) 0 2) 1 3) 5 4) 4 www.zioneducationonline.com 9899758900, 9891638000 Leading institute for MBA/GMAT/GRE/BBA/BCA/Hotel Mgt/Law entrance exams Centres at: North Campus, Pitampura, Rajinder Nagar, Noida, Greater Noida, Indirapuram(Ghaziabad)
  • 13. Q48. In a cricket match between India and Pakistan, 5 batsmen scored centuries. If 11 batsmen from each team batted, how many different sets of batsmen could have scored centuries? 1) 25132 2) 26434 3) 26334 4) 29262 5) None of these Q49. Of all the dresses she has, Kiza wears only the Red one, the green one and the Blue one to office. She dresses up in the following manner: She does not wear Red dress on any 2 consecutive days. She wears Green dress only if she has worn the blue dress on the previous day. In how many ways can she dress up for office on the 5 working days Monday to Friday, if she wants to wear the red dress at least once? 1) 36 2) 33 3) 45 4) 21 5) None of these Q50. 6 countries send their Prime ministers and Presidents for a UN conclave. In a round of 6 discussions, held at the conclave, each discussion was between a Prime minister and a President both from different countries. Find the total number of ways in which this conclave could have been conducted? 1) 984 2) 265 3) 1412 4) 1628 4) None of these. Q51. Ankit and Arpit started a cycle race from A to reach B. Arpit cycled at ¾th of Ankit’s speed. Ankit broke some rules and as a penalty had to go back to A after covering half the distance and then resumed the race. How long did Arpit take to finish the race if he won by 10 minutes? 1) 20 min 2) 25 min 3) 45 min 4) 80 min Directions for Ques 7 and 8. In a survey conducted on a group of 366 women over what they like about men, it was found that out of the total women who liked men for their sense of humour, 30% liked men with a good bank balance. A total of 90 women liked men for their bank balance only and the number of women liking men for their sense of humour and good looks was same as the number of women liking men for their good looks and bank balance. The ratio of the number of women liking men for their sense of humour and for their good looks was 2:3. No women liked men for all the 3 qualities. Q52. If the number of women liking men for their bank balance and sense of humour was 1.5 times the number of women liking men for their sense of humour and good looks, what was the number of women who liked men for their good looks only? 1) 86 2) 100 3) 120 4) 132 5) None of these. Q53. If the number of women who liked men only for their sense of humour was 24, what was the number of women who liked men for their good looks and bank balance? 1) 73 2) 74 3) 265 4) Cannot be determined 5) None of these. Q54. (100)! Is written in base 21. How many Zeroes will it have on the right of 1st non-zero digit from the right? 1) 24 2) 48 3) 12 3) 14 5) 16 Q55. Zion Education, a CAT coaching institute, has a weird teacher, who teaches Maths through games. He writes two different linear expressions on 2 chits, namely (7 – x) and ( 3 + x). Everyday he asks 2 students to chose one chit each and asks a third one to assign an integral value to the variable x. The student who gets the maximum value for the expression on his chit, for this value of ‘x’, gets a number of chocolates equal to the value of the expression. What is the minimum number of chocolates the teacher would have to give away at the end of 7 days, if the 3rd student assigns a different value to x everyday? 1) 35 2) 47 3) 56 4) 21 5) None of these www.zioneducationonline.com 9899758900, 9891638000 Leading institute for MBA/GMAT/GRE/BBA/BCA/Hotel Mgt/Law entrance exams Centres at: North Campus, Pitampura, Rajinder Nagar, Noida, Greater Noida, Indirapuram(Ghaziabad)
  • 14. Q56. 3 friends Magnum, Maggi and Maggu rolled a dice 3 times each. The outcomes were such that: i) For each of them, the 3 outcomes were distinct. ii) For all of them, the sum of the outcomes was the same and was more than 10. iii) Only one outcome was the same between any two of the 3 friends. Which of the following cannot be the common outcome between any 2 of the 3 friends? 1) 4 2) 2 3) 6 4) 3 5) None of these. Q57. Rituanjay marks up his goods by x% and then gives a discount of y% and thus makes a profit of 53%. If he had marked his goods up by y% and then given a discount of x%, then he would have suffered a loss of 77%. What is the value of x? The cost price of goods is Rs.100 1) 45 2) 15 3) 80 4) Cannot be determined 5) None of these. Q58. 2 sqaures are selected at random on a 8 x 8 chess board. What is the probability that they have a side in common. 1) 1/6 2) 1/12 3) 1/18 4) 1/9 5) It’s not possible Q59. A merchant navy ship with 9 different holes vertically aligned in its hull springs a leak in any of it’s 3 consecutive holes if 3 consecutive holes are closed. Find the number of ways in which it can spring a leak. 1) 10 2) 18 3) 17 4) 20 5) None of these Q60. Kiza and Fiza are twins. They are going for their CAT classes at Zion Education, which is 20 kms away from their home. They have 1 bicycle but they hate travelling with each other as people misunderstand Kiza for Fiza and Fiza for Kiza. They decide that Kiza will walk first and Fiza will go by bicycle and will leave at Delhi University Metro Station, where from Kiza will pick the bicycle and will go to Zion. Kiza walks at 5 k/h and cycles at 12 k/h while Fiza walks at 4 k/h and cycles at 10 k/h. They started from their house at the same time. How far should be Delhi University Metro Station from their house given that Kiza and Fiza reach Zion at the same time. 1) 7.5 km 2) 10 km 3) 12.5 km 4) 14.5 km 5) Data Insufficient Q61. 2 cards are drawn at random from a standard pack of 52 cards. What is the probability that 1 of the cards is King and another Hearts? 1) 1/13 2) C(4, 1) * C(13, 1) / C(52, 2) 3) 1/26 4) Cannot be determined 5) None of these Q62. Given that LogX (LogY (Logz p))) = 0, where each of x, y and z can assume values among 3, 27 and 81 only. If the product of all possible values of ‘p’ is represented in the form of 3^n, then what is the value of ‘n’? 1) 400 2) 480 3) 520 4) 360 4) 380 Q63. In a regular hexagon having a diagonal of length 56 cm, how many equilateral triangles of side 4 cm can one fit provided no triangle will overlap or intersect with each other? www.zioneducationonline.com 9899758900, 9891638000 Leading institute for MBA/GMAT/GRE/BBA/BCA/Hotel Mgt/Law entrance exams Centres at: North Campus, Pitampura, Rajinder Nagar, Noida, Greater Noida, Indirapuram(Ghaziabad)
  • 15. 1) 294 2) 60 3) 90 4) 45 5) None of these Q64. 2^100 – 1 is written in base 2. How many 1’s will it have? 1) 95 2) 100 3) 101 4) 105 5) None of these Q65. D, E and F are the points on sides BC, CA and AB of triangle ABC such that BD = DC; CE = 2 EA; AF = 3 FB. A(Tri DEF) = 1) ½ A( ABC) 2) ¾ A( ABC) 3) 7/24 A( ABC) 4) ½ A( ABC) Q66. Kripa singh uses a faulty weighing machine. The left pan of his beam balance is 200 gms heavier then the right pan. He uses the same beam balance while buying as well as selling. Assuming that he uses the beam balance logically, what is his overall profit percent. 1) 16.66% 2) 20% 3) 44.44% 4) 50% 5) cannot be determined 67. If N = (2012 + 201) – (1992 + 199), then what is the remainder when N is divided by 399? 1. 0 2. 2 3. 4 4. 399 5. None of these 68. [( )] What is the remainder when 257 + 338 2 is divided by 17? 1. 0 2. 1 3. 2 4. 3 5. 4 69. If any 4 whole numbers are chosen randomly and multiplied to each other, then what is the probability that the last digit of the product thus obtained is 1, 3, 7 or 9? 4 4 4 5 2 1 1 1 2 1.   2.   3. 5  4.   5. None of these 5 2 2 55 70. Let {an} be strictly increasing sequence of positive integers such that a2 = 2, amn = am an for m, n co prime. What is the value of a100=? 1. 99 2. 100 3. 101 4. Indeterminable 5. None of these www.zioneducationonline.com 9899758900, 9891638000 Leading institute for MBA/GMAT/GRE/BBA/BCA/Hotel Mgt/Law entrance exams Centres at: North Campus, Pitampura, Rajinder Nagar, Noida, Greater Noida, Indirapuram(Ghaziabad)